Can you help me please please correct my

Answers

Answer 1

Answer:

a = m/c

~~~~~~~~~~~~~~~~~~~~~~~

m=ca

divide both sides by c

you get a = m/c

Step-by-step explanation:

Answer 2

Answer:

a = m/c

Step-by-step explanation:

m = ca

divide both sides by c

m/c = ca/c

m/c = a

:.a = m/c


Related Questions

F={(-1, 2), (3, 2), (4,2), (0, 2)} Is Fa function and why/why not?

Answers

Answer:

yes it is a function         A

Step-by-step explanation:

A ball is thrown vertically with a velocity of18 m/s. It’s height, h, in meters above the ground after t seconds is given by the equation: h= -5t2+10t+35. Algebraically, determine the following.
Find The maximum height of the ball and the time it takes to reach that height
The time it takes the ball to hit the ground.

PLEASE HELP!

Answers

Answer:

Step-by-step explanation:

First of all, something is wrong with either the wording in the problem or the equation that you wrote; if the upward velocity is 18, we should see 18t in the equation, not 10t. I solved using 10t.

To find the max height of the ball and the time it took to get there, we need to complete the square on this quadratic and solve for the vertex. That will give us both of those answers in one!

To complete the square, set the quadratic equal to 0 and then move over the constant, like this:

[tex]-5t^2+10t=-35[/tex]  The rule is that we have to have a 1 as the leading coefficient, and right now it's a -5, so we factor that out, leaving us with:

[tex]-5(t^2-2t)=-35[/tex] and now we are ready to begin the process to complete the square.

The rule is: take half the linear term, square it, and add it to both sides. Our linear term is a -2 (from the -2t); half of -2 is -1, and -1 squared is 1. We add in a one to both sides. BUT when we put the 1 into the set of parenthesis on the left, we didn't just add in a 1, we have that -5 out front that is a multiplier. That means that we actually added in a -5 after it's all said and done.

[tex]-5(t^2-2t+1)=-35-5[/tex] and we'll clean that up a bit. The right side is easy, that's a -40. The left side...not so much.

The reason we complete the square is to put this quadratic into vertex form. Completing the square creates a perfect square binomial on the left, which for us is, along with the simplification on the right:

[tex]-5(t-1)^2=-40[/tex]

Lastly, we move the -40 back over by adding and setting the quadratic back to equal y:

[tex]-5(t-1)^2+40=y[/tex] and we see that the vertex is (1, 40). That translates to a height of 40 meters at 1 second after launch. That's the vertex which, by definition, is the max or min of the parabola. Because our parabola is negative, the vertex for us is a max.

To find out how long it takes the ball to hit the ground, set the quadratic equal to 0 and factor however it is you are currently doing this in class. You can continue to factor from the vertex form we have the equation in if you'd like. Let's do that, since we are already most of the way there. Begin here:

[tex]-5(t-1)^2=-40[/tex] and divide both sides by -5 to get

[tex](t-1)^2=8[/tex] and take the square root of both sides to "undo" that squaring on the left:

t - 1 = ±√8. Now add 1 to both sides to isolate the t:

t = 1 ± √8. In decimal form:

t = 1 + √8 is 3.828 seconds and

t = 1 - √8 is -1.828 seconds.

Since we all know that time will NEVER be a negative value, the time it takes the ball to hit the ground is 3.828 seconds.

Lucia gave Brenda half of the cash she had in the store counter at the end of the day. Brenda put the money in her purse and added one half of the amount she had from her own savings. Brenda had saved half of what Lucia earned in the store that day. Brenda took her purse and went to the dog shelter and brought food and treat packet for 5 dogs. Each packet cost her $15. How much money did Lucia earn at the store that day?

Answers

Answer:

$100

Step-by-step explanation:

The amount Lucia gave Brenda = Half the amount in the store counter

The amount Brenda added to the money Lucia gave her = Half the amount in her (Brenda) savings

The amount Brenda had saved = Half of the amount Lucia earned in the store that day

The number of food and treat packets Brenda bought = 5

The cost of each packet = $15

Let x represent the amount Lucia earned and let y represent the amount Brenda saved

We have;

x/2 = y

x/2 + y/2 = 15 × 5 = 75

Therefore, we get;

y + y/2 = 75

(3/2)·y = 75

y = 75 × 2/3 = 50

y = 50

From x/2 = y, we have;

x/2 = 50

x = 2 × 50 = 100

The amount Lucia earned in her store that day, x = $100

1 by 8 of the passenger of a train where children is there where 40 children traveling in the train on a Saturday how many Abbott were there in that that day?

Answers

Answer:

200 adults

Step-by-step explanation:

1/8 of the passengers (children) is 40

Total passengers in the train is 8/8

Therefore to get the total number of passengers is given by

(8/8) × 40 × (8/1)

= 240 passengers

Adults in the train = Passengers - Children

= 240 - 40

= 200 adults

If you roll a standard number cube 42 times, how many times do you expect the cube to show a five?
Round your answer to the nearest whole number if needed.

Answers

Answer:

5

Step-by-step explanation:

this is probability

probability of 5 occuring once is = 5/42=0.119

therefore, probability of 5 occuring in 42 times is;

42 x 0.119=4.998

approximately 5

There are 5 brown horses and 4 tan horses in a barn. Sonia will randomly select two horses to ride with her friend. What is the probability that
the first horse selected is tan and the second horse selected is brown?
20/81

5/18

2/9

1/20

Answers

Answer:

5/18

Step-by-step explanation:

The total number of horses present is 9

The probability that the first selected horse is tan is 4/9

So , now for the second choice , we are left with 8 total horses of which 5 is brown

The probability is 5/8

So the joint probability is the product of this two

That will be;

5/8 * 4/9 = 5/18

To make a disinfecting solution, Alana mixes 2 cups of bleach with 5 cups of
water. What is the ratio of water to the total amount of disinfecting solution?

Answers

Answer:

Step-by-step explanation:

Ok

Second time posting this. Please help!! :)

Answers

Answer:

Step-by-step explanation:

[tex]\frac{480+24(x-40)}{x}[/tex]

The numerator of the rational expression the money he earned for 'x' hours

The rate at which William is paid for each hour in excess of 40 hours 24.

x = 50 hours = (40 + 10 ) hours

The amount paid for excess 10 hours = 24 *10 = 240

Total amount earned for the week = 480 + 240 = 720

A diver begins at 140 feet below sea level. She descends at a steady rate of 7 feet per minute for 4.5 minutes. Then, she ascends 112.2 feet. What is her current depth?
Negative 549.3 feet
Negative 59.3 feet
59.3 feet
549.3 feet

Answers

Answer:

Step-by-step explanation:

starting point: 140 feet below sea level.=-140

she then decends= 7(4.5)=31.5

-140-31.5=-171.5

finally she ascends 112.2 feet

-171.5+112.2=-59.3 feet or 59.3 feet below sea level

Answer:

It's B

Step-by-step explanation:

Three events A, B and C are defined over a sample space, S. Events A and B are independent. Events A and C are mutually exclusive. Given that P(A)= 0.04, P(B)=0.25, P(C)=0.20 and P(B/C)=0.15. Find for P(C/B)

Answers

Answer:

[(b/c)=0.15)]

Step-by-step explanation:

Answer:  0.12

===================================================

Work Shown:

P(B/C) = P(B and C)/P(C) ... conditional probability formula

P(B and C) = P(C)*P(B/C)

P(B and C) = 0.20*0.15

P(B and C) = 0.03

------------

P(C/B) = P(C and B)/P(B) .... note the swap of B and C

P(C/B) = P(B and C)/P(B)

P(C/B) = (0.03)/(0.25)

P(C/B) = 0.12

------------

Extra notes:

The fact that events A and B are independent is not relevant.The fact A and C are mutually exclusive isn't used here either.This problem can be solved through Bayes' Theorem.Another alternative you can do is to set up a 3 by 3 contingency table to help solve this problem.

Solve the proportion.
14
y +9
-
10
15
y = [?]

Answers

Answer:

y=12

Step-by-step explanation:

14          10

----- =  --------

y+9         15

Using cross products

14*15 = 10 (y+9)

210 = 10(y+9)

Divide by 10

210/10 = 10(y+9)/10

21 = y+9

Subtract 9

21-9 = y

12 =y

A rectangle is twice as long as it is wide. Its width is 51⁄2 cm. Find the area of the rectangle.

Answers

Step-by-step explanation:

The are of rectangle is 1300.5cm2

In a jail cell, there are 5 Democrats and 6 Republicans. Four of these people will be
chosen for early release. How many 4-person groups are possible?

Answers

two because there is 11 people in total and you have two 4 person groups so that would leave 3 people left and you cant make a 4 person group w 3 people

Combination is a way of arranging the required items from a collection of items where the order of selection is not relevant.

If in a jail cell, there are 5 Democrats and 6 Republicans, the possible ways to form a group of 4 is 330.

The number of possible groups is 330.

What is combination?

Combination is a way of arranging the required items from a collection of items where the order of selection is not relevant.

It is given as:

[tex]^nC_r = \frac{n!}{r!(n-r)!}[/tex]

We have,

The total number of people:

5 Democrats and 6 Republicans = 5 + 6 = 11.

The number of persons in a group = 4.

The number of groups possible are:

= [tex]^{11}C_4[/tex]

= 11! / 4! ( 11-4)!

= 11! / 4! 7!

= 11 x 10 x 9 x 8 / 4 x 3 x 2

= 330

Thus,

The number of possible groups is 330.

Learn more about combination here:

brainly.com/question/2970011

#SPJ2

please help i will give brainliest (check both images)

Answers

Answer:

Amanda's box holds more popcorn.

Amanda's box holds 70 m³ more than Mary's.

Step-by-step explanation:

Find the volume of both boxes.

13.5 x 10 x 10 = 1350.

20 x 8 x 8 = 1280.

As clearly shown, Amanda's box holds more popcorn.

To find how much more, subtract 1,280 from 1,350 to get 70 m³.

Hope this helps!

If there is something wrong, please let me know.

Make a documentary on the “Journey of Coronavirus” OR Conservation of Plants and Animals. Give an appropriate name for your documentary.

PLSS REPLY FAST

Answers

Answer:

Explanation is as follows;

Step-by-step explanation:

Fundamentals to Plant and Animal Preservation

Plants and animals are the foundation of life on Earth; it is critical to protect them from extinction by maintaining a healthy population that contributes to global biodiversity, or the variety of flora and wildlife.

Everyone in the area must actively participate in planting additional trees and preventing countless trees from being chopped down. Deforestation is the practise of chopping down trees and making a forestland barren and unsuitable for animals and birds to live in.

Shelley drove from New Haven, Connecticut, to New York City in 90 minutes. Which equation relates the distance she traveled to her speed?

a.distance = speed + 90
b.distance = speed × 90
c.distance − speed = 90
d.distance × 90 = speed

Answers

Answer:

b

Step-by-step explanation:

d = r * t

None of them are really correct, but the closest one is B. Look at the equation. Rate is multiplied by time. The result is distance.

The 90 minutes should be changed to hours.  No speedometer on a car is rated in feet per second.

PLEASEEEE HELP, how do i draw this ?

Answers

Answer:

              /   -  -  - - - - - - -- - -  -

            /                                     \

        /                                           \

      /                                                \  _________

   /

_/

A lottery has a grand prize of 25and 1/2 million dollars. There are 5 people who have winning tickets and they share this prize equally. How many millions of dollars does each person get? Express the answer as a mixed number.

Answers

Answer:

5 1/10 million dollars

Step-by-step explanation:

25and 1/2 million dollars = 25 1/2 million dollars

= (25*2) + 1 / 2

= 50+1/2

= 51/2

Total prize = 51/2 million dollars

Number of people with winning ticket = 5

How many millions of dollars does each person get?

Amount of prize each person get = Total prize / Number of people with winning ticket

= 51 / 2 ÷ 5

= 51/2 × 1/5

= (51 * 1) / (2 * 5)

= 51/10

= 5 1/10 million dollars

Amount of prize each person get = 5 1/10 million dollars

What is the factored form of

Answers

Answer:

C

Step-by-step explanation:

You can distribute the values or even plug in x=2 and see which one matches.

Which expression is a sum of cubes

Answers

Answer: what is the rest of the question?

Step-by-step explanation:

need help solving the equation, trying to figure whether I multiply or divide.​

Answers

Answer:

m∠B = 43°

b = 26.1

c = 38.3

Step-by-step explanation:

By applying triangle sum in the given triangle,

47° + 90° + m∠B = 180°

137° + m∠B = 180°

m∠B = 43°

By applying sine ratio to the angle measuring 47°.

sin(47°) = [tex]\frac{\text{Opposite side}}{\text{Hypotenuse}}[/tex]

            = [tex]\frac{28}{c}[/tex]

c = [tex]\frac{28}{\text{sin}(47^{\circ})}[/tex]

c = 38.28

c ≈ 38.3

By applying cosine rule,

cos(47°) = [tex]\frac{\text{Adjacent side}}{\text{Hypotenuse}}[/tex]

             = [tex]\frac{b}{c}[/tex]

             = [tex]\frac{b}{38.3}[/tex]

b = 38.3[cos(47°)]

b = 26.1


The radius of the circle below intersects the unit circle at (3/5 • 4/5)What is the approximate value of e?

Answers

Answer:

do you have a pic of it but i think its 15.75

Step-by-step explanation:

What are the restrictions on the domain of the following function?
f(x)=x-3/(x-3)(x-4)(x-5)

Answers

Answer:

Step-by-step explanation:

C

Identify the segments that are parallel, if any, if ∠DCB and ∠CDA are supplementary.

Answers

Answer:

C

Step-by-step explanation:

We are given that

[tex]\angle[/tex]DCB and [tex]\angle[/tex]CDA are supplementary.

We have to identify the parallel segments if [tex]\angle[/tex]DCB and [tex]\angle[/tex]CDA are supplementary.

Supplementary angles: If the angles are supplementary then the sum of two angles is 180 degrees.

Therefore,

[tex]\angle DCB+\angle CDA=180^{\circ}[/tex]

Therefore, angle DCB and angle CDA are interior angles .

Because CA is a transversal line and sum of interior angles is 180 degrees.

When the sum of interior angles formed between two lines  and on same side of transversal line is 180  degrees. Then the lines are parallel.

Therefore, angle DCB and angle CDA are formed between line AD and BC.

Hence, AD is parallel to BC.

Option C is correct.

Answer:

c

Step-by-step explanation:

i had the same question

Which value is equivalent to x +4/4+x?
–4
–1
1
4

Answers

the answer is d. 4 i hope this helps:)

Hi! Please help me im really confused >.< tysm
g² - 4g = -3

Answers

Answer:

g = 1, 3

General Formulas and Concepts:

Pre-Algebra

Equality Properties

Algebra I

Terms/CoefficientsFactoringSolving quadratics

Step-by-step explanation:

Step 1: Define

Identify

g² - 4g = -3

Step 2: Solve for g

[Addition Property of Equality] Add 3 on both sides:                                     g² - 4g + 3 = 0Factor:                                                                                                                (g - 1)(g - 3) = 0Solve:                                                                                                                 g = 1, 3

How do I do this question

Answers

Answer:

Graph:

Solution: (1, 1)

Check:

y = -3x +4

1 = -3 +4

1 = 1

y = 3x -2

1 = 3 -2

1 = 1

Renata moved to her new home a few years ago. Back then, the young oak tree in her back yard was 190190190 centimeters tall. She measured it once a year and found that it grew at a constant rate. 333 years after she moved into the house, the tree was 274274274 centimeters tall.

Answers

Answer:

I know the answer...

Step-by-step explanation:

but i dont know the question (i also dont know how to comment so i have to do it as an answer)... thanks

Lyon walked 3/4 mile from school to the soccer field.

He then walked 1/6 mile home.


About how many miles did Lyon walk?

Answers

Answer:

11/12 miles Lyon walk.

Step-by-step explanation:

3/4 -> 9/12

1/6 -> 2/12

9 + 2 = 11

The solution is :about  11/12 miles Lyon walk.

What is addition?

Addition is a way of combining things and counting them together as one large group. ... Addition in math is a process of combining two or more numbers.

here, we have,

given that,

Lyon walked 3/4 mile from school to the soccer field.

He then walked 1/6 mile home.

now, we have to find that,

About how many miles did Lyon walk

so, we need to add the both distance

i.e.  3/4 + 1/6

so, we have,

3/4 -> 9/12

1/6 -> 2/12

so, we get,

9 + 2 = 11

i.e. 3/4 + 1/6

= 9/12 + 2/12

=11/12

Hence, about  11/12 miles Lyon walk.

To earn more on addition click:

brainly.com/question/29560851

#SPJ2

A tin of paint covers a surface area of 50m2. Each tin costs £5.60. The entire surface of a solid cylindrical rod with diameter 8m and height 12m needs to be painted. Find the minimum cost of painting the rod. (NO NEED FOR EXLENATION)​

Answers

the answer is in the picture

Other Questions
596 is divisible by 2?a.yesb.no solve A = 1/2bh, for b A company's managers should almost always give serious consideration to making significant adjustments in its camera/drone strategies and competitive approaches when: a. all or most of its competitors are using mostly different competitive approaches and therefore the marketplace is not big enough to accommodate all of the competitors. b. all or most of its competitors are using mostly copycat competitive approaches that make it difficult for any of these companies to capture sales volumes and revenues big enough to earn profits large enough to meet investor-expected EPS, ROE, and stock price appreciation targets. c. the number of camera and drone workstations the company has installed is NOT well above the industry-averages (as reported on pages 6 and 7 of the most recent Camera & Drone Journal). d. the company's market share for action cameras has not been the largest for two straight years and when its EPS and ROE have also not been the highest in the industry for two straight years. e. the company's operating profits per action camera sold are not substantially above the industry-average benchmarks in at least three geographic regions (as reported on p. 6 of the most recent Camera & Drone Journal), (4a)^2 without the exponents Find the surface area of each figure. Round your answers to the nearest tenth, if necessary What is the equation of the line that passes through (4,3) and (2, -1)? y = 4x -13y = 6x+4y = 2x-5y = 1/2 x -2 The nesxt simple quetions tell me Ans. 5) He crossed the broken bridge ____ warning.A) in spite ofB) in spite offC) onD) about , 6) The train ____ as fast as the bus.A) wentB) runningC) movesD) going. 7) He was seen _____ to the school.A) wentB) goingC) goneD) go Briefly describe theweaknesses of the Articlesof Confederation. The following data relate to direct materials for the month for the Hodge Wax Company: The standard costs for the work done was 5,900 pounds of wax at $9.50 per pound. The actual costs were 6,300 pounds at $9 per pound. What is the direct materials efficiency variance What role has the Supreme Court played in expanding rights for LGBTAmericans in the 21st century? A. The Supreme Court has ruled that churches must recognize same-sex marriage.B. The Supreme Court has struck down state and federal lawsbanning same-sex marriage.O C. The Supreme Court has ruled that a transgender woman must beemployed as a cabinet adviser.D. The Supreme Court has determined that gay pride parades mustbe held in all major cities. Which of the following is not a form of business combination?: (A) Acquisition. (B) S Corporation. (C) Conglomerate. (D) Merger. Beech Company produced and sold 105,000 units of its product in May. For the level of production achieved in May, the budgeted amounts were: sales, $1,300,000; variable costs, $750,000; and fixed costs, $300,000. The following actual financial results are available for May.ActualSales (105,000 units) $ 1,275,000Variable costs 712,500Fixed costs 300,000Prepare a flexible budget performance report for May.Beech CompanyFlexible Budget Performance ReportFor Month Ended May 31Flexible BudgetActual ResultsVarianceFavorable/Un fav.SalesVariable ExpenseContribution MarginFixed ExpenseIncome from Operations(Could you please show how to get the Flexible Budget) Which is the best estimate of -14 1/9 (-2 9/10) Can someone please help me with this?John is a 35-year-old male (he/him) administrative assistant who enjoys playing baseball on weekends. During one of their games, John began running from first to second base. As he pushed off to start running, John heard a pop and felt a sharp pain in his right calf as though he had just been kicked in the back of his leg. John fell to the ground and experienced significant pain when putting weight on his right leg. John tried to walk but had to be carried off the field and was taken to the emergency department. At the hospital, it was noted that John had significant swelling at the back of his right leg, had a palpable and visible deformity in the distal calf, and was only able to put small amounts of weight on his right leg. The emergency doctor squeezed the back of Johns right calf and noticed that the ankle would not plantarflex. An x-ray and blood work were ordered which came back without any significant findings.Case Study Questions:1. Explain the possible anatomical structures that were affected by the injury and how each relates to Johns presenting symptoms and test results (250 word maximum).2. Identify and explain three differential diagnoses (diseases or conditions that present similar signs and symptoms that could possibly account for the patient's symptoms) with appropriate justifications. After describing your three differential diagnoses, indicate the most likely diagnosis based on your analysis and provide rationale and support for your selection. 3. From an anatomy point of view:a) Propose a management plan for the most likely diagnosis. This should include the treatment options you would implement given your patient's presenting symptoms. You should provide short- and long-term goals for the patient and a timeline on when you would implement each treatment. There may be several possible treatments you could prescribe to your patient. Include support, justification, and evidence for your prescribed treatment(s) and management plan. b) Explain the effects of your management plan for your patient's condition. From the management plan you've proposed, what effects will each treatment have on the patient's anatomy? Include any positive and negative effects this plan may have on the patient. 9) Assume that the random variable X is normally distributed, with mean = 90 and standard deviation o = 12. Compute the probability P(57 < X < 105). A) 0.7888 B) 0.8944 C) 0.8914 D) 0.8819 What is literature? Predict the approximate Ksp of Cuz(AsO4)2 based on the measured potential of Cell 7. Use the equation given in the Background. a. 1 x 10^-35 b. 4 x 10^14 c. 5 x 10^-17 d. 2 x 10^-21 After running 3/4 of a mile tess has only run 1/3 how long is the race in miles but I want to know how you did it Please solve all questions. One angle of a triangle is equal to the sum of the remaining angles. If the ratio of measures of the renis 2:1, find the measures of the three angles of the triangle.